B 数学分析

时间序列分析用到了数学分析、复分析、实变函数、泛函分析、测度论、概率论、随机过程、数理统计、多元统计分析中的一些结果。 这里对一些数学知识进行整理。

B.1 极限

定理B.1 (分部求和公式) \(\{ x_k, k=m, m+1, \dots, n \}\), \(\{ y_k, k=m, m+1, \dots, n+1 \}\)是数列,则 \[\begin{aligned} \sum_{k=m}^n x_k (y_{k+1} - y_k) =& [x_n y_{n+1} - x_m y_m] - \sum_{k=m+1}^n y_k (x_k - x_{k-1}) \\ =& [x_n y_{n+1} - x_m y_m] - \sum_{k=m}^{n-1} y_{k+1} (x_{k+1} - x_{k}) \end{aligned}\]

证明\[\begin{aligned} \text{左边} =& \sum_{k=m}^n x_k y_{k+1} - \sum_{k=m}^n x_k y_k \\ \text{右边} =& x_n y_{n+1} - x_m y_m - \sum_{k=m+1}^n x_k y_k + \sum_{k=m+1}^n x_{k-1} y_k \\ =& x_n y_{n+1} + \sum_{s=m}^{n-1} x_s y_{s+1} - \sum_{k=m}^n x_k y_k \\ = & \sum_{s=m}^{n} x_s y_{s+1} - \sum_{k=m}^n x_k y_k \\ =& \text{左边} \end{aligned}\]

○○○○○○

定理B.2 (Kronecker引理) \(\{ x_n, n \in \mathbb N_+ \}\)是复数列, \(\sum_{n=1}^\infty x_n\)收敛到复数\(s\)。 实数列\(\{ b_n \}\)满足 \(0 < b_1 \leq b_2 \leq \dots\)\(\lim_{n\to\infty} b_n = \infty\), 则 \[ \lim_{n\to\infty} \frac{1}{b_n} \sum_{k=1}^n b_k x_k = 0 . \]

证明: 记\(S_n = \sum_{j=1}^n x_j\), \(S_0 = 0\), \(y_{n+1} = S_n\)。 由分部求和公式有 \[\begin{aligned} & \sum_{k=1}^n b_k x_k = \sum_{k=1}^n b_k (S_k - S_{k-1}) = \sum_{k=1}^n b_k (y_{k+1} - y_{k}) \\ =& b_n y_{n+1} - b_1 y_1 - \sum_{k=1}^{n-1} y_{k+1} (b_{k+1} - b_{k}) \\ =& b_n S_n - \sum_{k=1}^{n-1} S_{k} (b_{k+1} - b_{k}) \end{aligned}\] 于是 \[\begin{aligned} \frac{1}{b_n} \sum_{k=1}^n b_k x_k = S_n - \frac{1}{b_n} \sum_{k=1}^{n-1} (b_{k+1} - b_k) S_k \end{aligned}\] 由于\(S_n \to s\), \(\forall \varepsilon>0\), 存在\(N\)使得\(n \geq N\)\(| S_n - s | < \varepsilon/2\)。 将上式右边变成 \[\begin{aligned} & S_n - \frac{1}{b_n} \sum_{k=1}^{N-1} (b_{k+1} - b_k) S_k - \frac{1}{b_n} \sum_{k=N}^{n-1} (b_{k+1} - b_k) S_k \\ =& S_n - \frac{1}{b_n} \sum_{k=1}^{N-1} (b_{k+1} - b_k) S_k - \frac{1}{b_n} \sum_{k=N}^{n-1} (b_{k+1} - b_k) s - \frac{1}{b_n} \sum_{k=N}^{n-1} (b_{k+1} - b_k) (S_k - s) \\ =& S_n - \frac{1}{b_n} \sum_{k=1}^{N-1} (b_{k+1} - b_k) S_k - \frac{b_n - b_N}{b_n} s - \frac{1}{b_n} \sum_{k=N}^{n-1} (b_{k+1} - b_k) (S_k - s) \\ \end{aligned}\]\(n\to\infty\)时, 第一项和第三项分别趋于\(s\)\(-s\),可以消去; 第二项趋于0, 第四项的绝对值小于等于 \(\frac12\varepsilon \frac{b_n - b_N}{b_n} \leq \frac12\varepsilon\), 所以存在\(N_2>N\)使得\(n > N_2\)时四项之和绝对值小于\(\varepsilon\)。 Knonecker引理证毕。

○○○○○○

定理B.3 (Stolz定理) 设实数列\(\{ a_n \}\)\(\{ b_n \}\)满足

(1) \(\{b_n \}\)严格单调递增;

(2) \(\lim_{n\to\infty} b_n = +\infty\);

(3) \(\lim_{n\to\infty} \frac{a_{n+1} - a_{n}}{b_{n+1} - b_n} = L\) 有意义,\(L\)为有限实数、\(+\infty\)\(-\infty\)

则有

\[ \lim_{n\to\infty} \frac{a_n}{b_n} = \lim_{n\to\infty} \frac{a_{n+1} - a_{n}}{b_{n+1} - b_n} = L \]

这是与微积分中洛必达法则类似的数列极限定理。

证明: 当\(L\)为有限实数时, 由条件(3)和条件(1)可知, \(\forall \epsilon>0\), \(\exists N_1 > 0\), 当\(n > N_1\)\[ \left| \frac{a_{n+1} - a_{n}}{b_{n+1} - b_n} - L \right| < \epsilon \] 从而 \[ L - \epsilon < \frac{a_{n+1} - a_{n}}{b_{n+1} - b_n} < L + \epsilon \] \[ (L - \epsilon)(b_{n+1} - b_n) < a_{n+1} - a_{n} < (L + \epsilon)(b_{n+1} - b_n) \quad (*) \]

由条件(2), \(\exists N_2>N_1\), 当\(n > N_2\)\(b_n > \epsilon > 0\)

\(n>N_2\)时, 从\(N_2+1\)\(n\)对(*)式累加, 有 \[ (L-\epsilon)(b_{n+1} - b_{N_2+1}) < a_{n+1} - a_{N_2+1} < (L+\epsilon)(b_{n+1} - b_{N_2+1}) \] 于是 \[ L-\epsilon < \frac{a_{n+1} - a_{N_2+1}}{b_{n+1} - b_{N_2+1}} < L+\epsilon \]\(b_{n+1} > \epsilon > 0\),得 \[ L-\epsilon < \frac{\frac{a_{n+1}}{b_{n+1}} - \frac{a_{N_2+1}}{b_{n+1}}}{1 - \frac{b_{N_2+1}}{b_{n+1}}} < L+\epsilon \]\(n\to\infty\), 因为 \[ \lim_{n\to\infty} \frac{a_{N_2+1}}{b_{n+1}} = 0, \quad \lim_{n\to\infty} \frac{b_{N_2+1}}{b_{n+1}} = 0 \] 所以存在\(N_3 > N_2\)\(n > N_3\)\[ |L+\epsilon| \cdot \left| \frac{b_{N_2+1}}{b_{n+1}} \right| < \epsilon, \quad |L-\epsilon| \cdot \left| \frac{b_{N_2+1}}{b_{n+1}} \right| < \epsilon, \quad \left| \frac{a_{N_2+1}}{b_{n+1}} \right| < \epsilon \] 于是 \[ L - 2\epsilon < \frac{a_{n+1}}{b_{n+1}} - \frac{a_{N_2+1}}{b_{n+1}} < L + 2\epsilon \] \[ L - 3\epsilon < \frac{a_{n+1}}{b_{n+1}} < L + 3\epsilon \]\[ \left| \frac{a_{n+1}}{b_{n+1}} - L \right| < 3\epsilon \]\[ \lim_{n\to\infty} \frac{a_n}{b_n} = L . \]

\(L\)为无穷时的证明略。

○○○○○○

推论B.1 如果数列\(a_n \to 0\)(\(n\to\infty\)), 则 \[ \lim_{n\to\infty} \frac{1}{n} \sum_{i=1}^n a_i = 0 \]

证明: 由Stolz定理,记\(S_n = \sum_{i=1}^n a_i\),则 \[\begin{aligned} \lim_{n\to\infty} \frac{1}{n} \sum_{i=1}^n a_i =& \lim_{n\to\infty} \frac{S_n}{n} \\ =& \lim_{n\to\infty} \frac{S_n - S_{n-1}}{n - (n-1)} \\ =& \lim_{n\to\infty} a_n = 0 \end{aligned}\]

○○○○○○

B.2 微积分

定理B.4 (微积分基本定理) (1) 若\(f(x)\)是定义在\([a,b]\)上的Riemann可积函数且在\(x=x_0\)处连续, 则函数 \[\begin{aligned} F(x) = \int_a^{x} f(t)dt, \quad x \in [a,b] \end{aligned}\]\(x=x_0\)处可微且\(F'(x_0)=f(x_0)\)

(2) 若\(f(x)\)是定义在\([a,b]\)上的可微函数, \(f'(x)\)\([a,b]\)上是Riemann可积函数, 则\(f(x)\)是其导函数的不定积分: \[\begin{aligned} \int_a^x f'(t) dt = f(x) - f(a), \quad x \in [a,b] \end{aligned}\]

对Lebesgue积分也有类似结论。

定理B.5 (Lebesgue定理) \(f(x)\)是定义在\([a,b]\)上的单调上升实值函数, 则\(f(x)\)的不可微点集为零测集且有 \[\begin{aligned} \int_a^b f'(x) dx \leq f(b) - f(a) \end{aligned}\]

勒贝格积分与黎曼积分关系

黎曼积分是按照\(x\)的区间进行分割, 当细分小区间长度趋于零时的极限(如果存在)。 勒贝格积分是按照函数值\(y\)的区间进行分割, 用简单函数的积分逼近一般函数的积分。

定理B.6 对闭区间\([a,b]\)上的有界函数\(f\), 如果黎曼可积, 则\(f\)必为Borel可测函数且勒贝格可积, 积分值相等。

定理B.7 对闭区间\([a,b]\)上的有界函数\(f\)\(f\)黎曼可积的充分必要条件是\(f\)\([a,b]\)中的不连续点组成的集合为勒贝格零测集。

推论\([a,b]\)上仅有有限个不连续点的函数是黎曼可积的, 也是勒贝格可积的, 两种积分相等。

定义B.1 (有界变差函数) \(f(x)\)是定义在\([a,b]\)上的实值函数, 作分划\(\Delta_t\): \(a=x_0 < x_1 < \dots < x_n = b\) 以及相应的和 \[\begin{aligned} \nu_\Delta = \sum_{i=1}^n | f(x_i) - f(x_{i-1}) | \end{aligned}\]\[\begin{aligned} \bigvee_a^b(f) = \sup \{ \nu_\Delta: \Delta \text{为$[a,b]$的任一分划} \} \end{aligned}\] 并称它为\(f\)\([a,b]\)上的全变差。若 \[\begin{aligned} \bigvee_a^b(f) < +\infty \end{aligned}\] 则称\(f(x)\)\([a,b]\)上的有界变差函数, 其全体记为\(BV([a,b])\)

有界变差函数有界, \(BV([a,b])\)构成一个线性空间。

B.3 数值级数

\(\{ a_n, n = 1,2,\dots \}\)为实数列, 考虑\(\sum_{n=1}^{\infty} a_n\)。 称 \[ S_n = \sum_{i=1}^n a_i \] 为部分和序列。 如果\(S_n\)有实数值极限\(S\), 则称级数\(\sum_{n=1}^{\infty} a_n\)收敛到\(S\); 如果\(S_n\)极限为\(+\infty\)\(-\infty\), 则称级数\(\sum_{n=1}^{\infty} a_n\)发散到\(+\infty\)\(-\infty\); 如果\(S_n\)极限不存在, 则称级数\(\sum_{n=1}^{\infty} a_n\)发散。

如果\(\sum_{n=1}^{\infty} |a_n|\)收敛到有限值, 则称级数\(\sum_{n=1}^{\infty} a_n\)绝对收敛, 绝对收敛推出收敛。

如果级数\(\sum_{n=1}^{\infty} a_n\)收敛, 则\(\lim_{n \to \infty} a_n = 0\)

对正项级数\(\sum_{n=1}^{\infty} a_n\)\(\sum_{n=1}^{\infty} b_n\), 如果\(\lim_{n\to\infty} \frac{a_n}{b_n}\)为有限的非零实数值, 即\(a_n\)\(b_n\)同阶, 则两个级数同时收敛或者同时发散。

达朗倍尔判别法: 设\(\sum_{n=1}^{\infty} a_n\)是正项级数, 若 \[ \varlimsup_{n\to\infty} \frac{a_{n+1}}{a_n} = r < 1, \]\(\sum_{n=1}^{\infty} a_n\)收敛; 若 \[ \varliminf_{n\to\infty} \frac{a_{n+1}}{a_n} = r > 1, \]\(\sum_{n=1}^{\infty} a_n\)发散; \(r=1\)时不能判断。

哥西判别法: 设\(\sum_{n=1}^{\infty} a_n\)是正项级数, 若 \[ \varlimsup_{n\to\infty} a_n^{1/n} = \rho, \] 则当\(\rho<1\)时级数收敛, 当\(\rho>1\)时级数发散。

如果级数\(\sum_{n=1}^{\infty} a_n\)绝对收敛, 则任意改变求和次序, 级数仍绝对收敛, 且收敛到相同值; 否则, 改变求和次序可能发散或收敛到不同的结果。

二重级数: 对数列\(\{a_{ij}, i=1,2,\dots, j=1,2,\dots\}\), 令\(S_i = \sum_{j=1}^\infty a_{ij}\), 若每个\(S_i\)收敛, 且\(\sum_{i=1}^\infty S_i\)收敛, 则级数\(\sum_{i=1}^\infty \sum_{j=1}^\infty a_{ij}\)收敛到\(\sum_{i=1}^\infty S_i\)

如果其中\(\sum_{i=1}^\infty \sum_{j=1}^\infty |a_{ij}| < \infty\), 则可交换次序 \[ \sum_{i=1}^\infty \sum_{j=1}^\infty a_{ij} = \sum_{j=1}^\infty \sum_{i=1}^\infty a_{ij} . \]

级数乘法: 设级数\(\sum_{n=1}^{\infty} a_n\)\(\sum_{n=1}^{\infty} b_n\)至少有一个绝对收敛, 则 \[ \left( \sum_{n=1}^{\infty} a_n \right) \left( \sum_{n=1}^{\infty} b_n \right) = \sum_{n=1}^{\infty} c_n , \] 其中 \[ c_n = \sum_{i=1}^{n} a_i b_{n+1-i} . \]

常用求和公式:

\[\begin{aligned} 1+2+3+\dots+n =& \sum_{k=1}^n k = \frac{1}{2} n (n+1) . \\ 1^2 + 2^2 + 3^2 + \dots + n^2 =& \sum_{k=1}^n k^2 = \frac{1}{6} n(n+1)(2n+1) .\\ 1^3 + 2^3 + 3^3 + \dots + n^3 =& \sum_{k=1}^n k^3 = \left( \frac{1}{2} n (n+1) \right)^2 .\\ 1^4 + 2^4 + 3^4 + \dots + n^4 =& \sum_{k=1}^n k^4 = \frac{1}{30} n(n+1)(2n+1)(3n^2+3n-1) . \end{aligned}\]

B.4 函数项级数

\(f_n(x)\)是定义在区间\(I\)上的函数, \(n=1,2,\dots\),称\(\{ f_n(x), n=1,2,\dots \}\)为函数序列。 如果在\(I\)的一个非空子集\(I_1\)\[ \lim_{n\to\infty} f_n(x) = f(x), \ \forall x \in I_1, \] 则称\(f(x)\)\(I_1\)上是函数序列的极限函数。

考虑函数级数\(\sum_{n=1}^\infty u_n(x)\)\(u_n(x)\)是区间\(I\)上的函数, 如果其部分和序列 \[ S_n(x) = \sum_{i=1}^n u_i(x) \]\(I_1 \subset I\)中收敛到极限函数\(S(x)\), 则称级数\(\sum_{n=1}^\infty u_n(x)\)\(I_1\)中收敛到\(S(x)\)

使得级数\(\sum_{n=1}^\infty u_n(x)\)收敛的点\(x\)称为收敛点, 否则称为发散点。 所有收敛点组成的集合称为收敛区域, 所有发散点组成的集合称为发散区域。

如果\(\sum_{n=1}^\infty |u_n(x)|\)\(I_1\)中收敛, 则称\(\sum_{n=1}^\infty u_n(x)\)\(I_1\)中绝对收敛, 绝对收敛推出收敛。

级数\(\sum_{n=1}^\infty u_n(x)\)与极限\(\lim_{n\to\infty} \sum_{i=1}^n u_i(x)\)是同一问题。 对其中函数的微分、积分、极限等操作能否与求和或者极限运算交换次序? 在一致收敛条件下可以。

一致收敛: 设\(f_n(x)\)在区间\(I_1\)有极限函数\(f(x)\), 如果任给\(\epsilon>0\),都存在一个不依赖于\(x\)的正整数\(N\), 当\(n \geq N\)时, 对任意\(x \in I_1\)都有 \[ |f_n(x) - f(x)| < \epsilon, \] 则称\(f_n(x)\)在区间\(I_1\)一致收敛到\(f(x)\)。 类似地, 如果级数的部分和序列一致收敛, 则称级数一致收敛。

\(f_n(x)\)在区间\(I_1\)一致收敛到\(f(x)\), 当且仅当 \[ \lim_{n\to\infty} \sup_{x \in I_1} |f_n(x) - f(x)| = 0 . \]

对函数级数\(\sum_{n=1}^\infty u_n(x)\), 如果\(\sum_{i=n+1}^\infty u_i(x)\)一致收敛到0, 则函数级数一致收敛。

极限次序交换: 设函数\(f_n(x)\), \(n=1,2,\dots\)定义在\([a,b]\)上, \(x_0 \in [a,b]\)\(f_n(x)\)\([a,b] \backslash \{x_0\}\)上一致收敛到\(f(x)\), 设 \(\lim_{x \to x_0} f_n(x)\)存在, 则 \[ \lim_{x\to x_0} \lim_{n\to \infty} f_n(x) = \lim_{n\to \infty} \lim_{x\to x_0} f_n(x) . \]

极限与求和号交换次序: 设函数\(u_n(x)\), \(n=1,2,\dots\)定义在\([a,b]\)上, \(x_0 \in [a,b]\)\(\sum_{n=1}^\infty u_n(x)\)\([a,b] \backslash \{x_0\}\)上一致收敛到\(S(x)\), 设 \(\lim_{x \to x_0} u_n(x)\)存在, 则 \[ \lim_{x \to x_0} \sum_{n=1}^\infty u_n(x) = \sum_{n=1}^\infty \lim_{x \to x_0} u_n(x) . \]

如果\(f_n(x)\)是闭区间\([a,b]\)上的连续函数, \(f_n(x)\)\([a,b]\)上一致收敛到\(f(x)\), 则\(f(x)\)也是闭区间\([a,b]\)上的连续函数。

如果\(u_n(x)\)是闭区间\([a,b]\)上的连续函数, \(\sum_{n=1}^\infty u_n(x)\)在在\([a,b]\)上一致收敛到\(S(x)\), 则\(S(x)\)也是闭区间\([a,b]\)上的连续函数。

如果\(u_n(x)\)是开区间\((a,b)\)上的连续函数, \(\sum_{n=1}^\infty u_n(x)\)\((a,b)\)内每一个闭区间上都一致收敛到\(S(x)\), 则\(S(x)\)也是开区间\((a,b)\)上的连续函数。

积分号下取极限: 设\(f_n(x)\)是闭区间\([a,b]\)上的连续函数, \(f_n(x)\)\([a,b]\)上一致收敛到\(f(x)\), 则 \[ \lim_{n\to\infty} \int_a^b f_n(x) \,dx = \int_a^b \lim_{n\to\infty} f_n(x) \,dx . \]

积分与求和号交换次序: 设\(u_n(x)\)是闭区间\([a,b]\)上的连续函数, 级数\(\sum_{n=1}^\infty u_n(x)\)在在\([a,b]\)上一致收敛到\(S(x)\), 则 \[ \int_a^b \sum_{n=1}^\infty u_n(x) \,dx = \sum_{n=1}^\infty \int_a^b u_n(x) \,dx . \]

微分与求和号交换次序: 设\(u_n(x)\)在闭区间\([a,b]\)上可微, \(\sum_{n=1}^\infty u_n'(x)\)一致收敛, 且\(\sum_{n=1}^\infty u_n(x)\)至少在某一个点\(x_0\)上收敛, 则\(\sum_{n=1}^\infty u_n(x)\)\([a,b]\)上一致收敛, 且 \[ \left( \sum_{n=1}^\infty u_n(x) \right)' = \sum_{n=1}^\infty u_n'(x) . \]

B.5 幂级数

形如 \[ \sum_{n=0}^\infty a_n (x - x_0)^n \] 的函数项级数称为幂级数, 其中\(x_0\)是任意给定实数, \(\{ a_n \}\)是实数列。 实际上只要考虑 \[\begin{align} \sum_{n=0}^\infty a_n x^n . \tag{B.1} \end{align}\]

幂级数(B.1)的收敛区域只有如下三种情况:

  1. 整个实数轴;
  2. 关于原点对称的有限区间\((-R, R)\),可含端点;
  3. 只在\(x=0\)处收敛。

\[ \rho = \varlimsup_{n\to\infty} |a_n|^{1/n}, \]\(0 \leq \rho < \infty\)时, 幂级数(B.1)\(|x| < \frac{1}{\rho}\)绝对收敛; 当\(0 < \rho < \infty\)\(|x|>\frac{1}{\rho}\)时, 幂级数(B.1)发散。 称\(R = \frac{1}{\rho}\)为幂级数的收敛半径, \((-R, R)\)为幂级数的收敛区间。 当\(\rho=0\)时,收敛区间是\((-\infty, \infty)\); 当\(\rho=\infty\)时, 仅在\(x=0\)处收敛。 收敛区间端点处是否收敛不确定。

若幂级数(B.1)\(x = x_1 \neq 0\)处收敛, 则它在\(|x| < |x_1|\)处绝对收敛; 如果级数在\(x = x_0\)处发散,则它在\(|x| > |x_0|\)处也发散。

如果 \[ \lim_{n\to\infty} \frac{|a_{n+1}|}{|a_n|} = l, \] 则幂级数(B.1)的收敛半径\(R = 1/l\)(包括\(l=0\)\(l=\infty\)的情况)。

设幂级数(B.1)的收敛半径\(R>0\), 则对任意\(0 < r < R\),级数在\([-r, r]\)上一致收敛, 称为在\((-R, R)\)内闭一致收敛。

幂级数(B.1)在收敛区间\((-R, R)\)内是连续函数。

幂级数微分: 幂级数(B.1)在收敛区间\((-R, R)\)内可微, 且微分与求和号可交换: \[ \left(\sum_{n=0}^\infty a_n x^n \right)' = \sum_{n=1}^\infty n a_n x^{n-1} , \] 右边的级数与(B.1)有相同的收敛半径。

幂级数积分: 设幂级数(B.1)收敛半径\(R > 0\), 则积分号与求和号可交换: \[ \int_0^x \sum_{n=0}^\infty a_n t^n \,dt = \sum_{n=0}^\infty \frac{a_n}{n+1} x^{n+1} , \] 右边的级数与(B.1)有相同的收敛半径。

泰勒展开: 设函数\(f(x)\)\(I = (x_0 - \delta, x_0 + \delta)\)上有任意阶导数, 且存在正常数\(M\)使得 \[ | f^{(n)}(x) | \leq M^n , \ \forall x \in I, \ n=1,2,\dots, \] 则对\(x \in I\)\[ f(x) = \sum_{n=0}^\infty \frac{f^{(n)}(x_0)}{n!} (x - x_0)^n . \]

B.6 傅立叶级数

考虑复数域上的希尔伯特空间 \(L^2[-\pi, \pi] = (L^2[-\pi, \pi], \mathscr B, U)\), 其中\(\mathscr B\)\([-\pi,\pi]\)上的Borel集组成的\(\sigma\)域, \(U\)\([-\pi,\pi]\)上的Lebegue测度。 定义内积为 \[\begin{aligned} <f, g> = E(f \bar g) = \frac{1}{2\pi} \int_{-\pi}^\pi f(x) \bar g(x) dx. \end{aligned}\] 这时\(\{ e_n = e^{inx}, n \in \mathbb Z \}\)构成标准正交基。 如果\(f \in L^2[-\pi, \pi]\)\[\begin{aligned} < f, e_j > = 0, \ \forall j \in \mathbb Z \end{aligned}\]\[\begin{aligned} f(x) = 0, \ \text{a.e.} \end{aligned}\]

\(f \in L^2[-\pi, \pi]\), 令 \[\begin{aligned} S_n f = \sum_{j=-n}^n <f, e_j> e_j, \end{aligned}\] 其中 \[\begin{aligned} <f, e_j> = \frac{1}{2\pi} \int_{-\pi}^\pi e^{-ijx} f(x) dx \end{aligned}\] 叫做\(f\)的Fourier系数, Fourier系数列必平方可和。 \(S_n f\)叫做\(f\)\(n\)阶Fourier逼近, \(S_n f\)\(f\)\(\overline{\mbox{sp}}\{e_j, |j| \leq n \}\)上的投影。

\(S_n f\)均方极限存在且等于\(f\)\(S_n f\)的极限写成函数级数 \[\begin{aligned} S f = \sum_{j=-\infty}^\infty <f, e_j> e_j. \end{aligned}\]

\[\begin{aligned} L^2[-\pi, \pi] = \overline{\mbox{sp}}\{ e_j, j \in \mathbb Z \}. \end{aligned}\]

\[\begin{aligned} \| f \|^2 = \sum_{j=-\infty}^\infty | < f, e_j > |^2. \end{aligned}\]

\[\begin{aligned} <f, g> = \sum_{j=-\infty}^\infty <f, e_j> \cdot \overline{<g, e_j>}. \end{aligned}\]

\(f(x)\)是以\(2\pi\)为周期的连续函数, 则任给\(\varepsilon>0\), 存在三角多项式 \[\begin{aligned} T_n(x) = \frac{a_0}{2} + \sum_{j=1}^{n} \left\{ a_j \cos(jx) + b_j \sin(jx) \right\} \end{aligned}\] 使得 \[\begin{aligned} |f(x) - T_n(x)| < \varepsilon,\ \forall x \in (-\infty,\infty) \end{aligned}\] 事实上, \[\begin{aligned} n^{-1}(S_0 f + S_1 f + \dots S_{n-1} f) \to f \end{aligned}\]\([-\pi,\pi]\)一致收敛\((n\to\infty)\)

\(f(x)\)是以\(2\pi\)为周期的连续函数, 且\(f' \in L^2[-\pi,\pi]\), 则\(S_n f\)不仅均方收敛到\(f\), 而且绝对一致收敛到\(f\)。 (见(Brockwell and Davis 1987)§2.8, §2.11)。

对于以\(2\pi\)为周期的函数\(f(x)\), 如果在\([-\pi, \pi]\)上可积(有瑕点时绝对可积), 则可以计算 \[\begin{aligned} a_n =& \frac{1}{\pi} \int_{-\pi}^{\pi} f(x) \cos(nx) dx, \ n=0, 1, 2, \dots \\ b_n =& \frac{1}{\pi} \int_{-\pi}^{\pi} f(x) \sin(nx) dx, \ n=1, 2, \dots \end{aligned}\] 并形式地写出函数级数 \[\begin{aligned} \frac{a_0}{2} + \sum_{n=1}^{\infty} \left\{ a_n \cos(nx) + b_n \sin(nx) \right\} \end{aligned}\] 但不能保证级数收敛且收敛到\(f(x)\)

如果\(f(x)\)\(x=x_0\)处满足\(\alpha\)级(\(0<\alpha \leq 1\))李普希兹条件: \[\begin{aligned} | f(x_0 \pm t) - f(x_0) | \leq L t^\alpha, \ 0<t\leq \delta \end{aligned}\] (其中\(L>0, \delta>0\)), 则\(f(x)\)的傅立叶级数在\(x_0\)处收敛到\(f(x)\)

\(f(x)\)\([a,b]\)逐段可微(除了有限个点外可微,在这些点上有左右导数), 则其傅立叶级数在每个\(x=x_0\)处均收敛到 \[\begin{aligned} S_0 = \frac{f(x_0+0) + f(x_0-0)}{2} \end{aligned}\] 当然,除去不可微的有限个点之外都收敛到\(f(x_0)\)

若对点\(x_0\)存在\(h>0\)使得\(f(x)\)\([x_0 - h, x_0]\)\([x_0, x_0 + h]\)分别单调, 则\(f(x)\)的傅立叶级数在\(x_0\)收敛到 \[\begin{aligned} \frac{f(x_0+0) + f(x_0-0)}{2} \end{aligned}\]

\(f(x)\)逐段单调,则其傅立叶级数对任意\(x_0\)均收敛到 \[\begin{aligned} \frac{f(x_0+0) + f(x_0-0)}{2} \end{aligned}\]

\(f(x)\)在区间\([-\pi,\pi]\)上平方可积, 则\(\forall \varepsilon>0\), 存在三角多项式\(T(x)\)使得 \[\begin{aligned} \int_{-\pi}^\pi | f(x) - T(x) |^2 dx < \varepsilon \end{aligned}\]

\(f(x)\)在区间\([-\pi,\pi]\)上黎曼可积或在广义积分意义下平方可积, 设\(S_n(f,x)\)为其傅立叶级数的部分和, 则 \[\begin{aligned} \lim_{n\to\infty} \int_{-\pi}^\pi | f(x) - S_n(f,x) |^2 dx = 0 \end{aligned}\]

B.7 参变积分

定理B.8 (参变积分连续性(一)) 设二元函数\(f(x,y)\)\([a,b] \times [\alpha, \beta]\)上的连续函数, 则 \[ g(x) = \int_{\alpha}^{\beta} f(x, y) \,dy \]\([a,b]\)上的连续函数。

推论(积分号下取极限) 在定理条件下对\(x_0 \in [a,b]\)\[ \lim_{x\to x_0} \int_{\alpha}^{\beta} f(x, y) \,dy = \int_{\alpha}^{\beta} \lim_{x\to x_0} f(x, y) \,dy . \]

如果是广义积分或者瑕积分则需要更强的条件。

定理B.9 (参变积分连续性(二)) 设二元函数\(f(x,y)\)\([a,b] \times [\alpha, \beta]\)上的连续函数, \(\phi(x)\), \(\psi(x)\)\([a, b]\)上的连续函数且取值于\([\alpha,\beta]\), 则 \[ g(x) = \int_{\phi(x)}^{\psi(x)} f(x, y) \,dy \]\([a,b]\)上的连续函数。

定理B.10 (积分号下求导) \(f(x,y)\)\(\frac{\partial f(x,y)}{\partial x}\)都是\([a,b] \times [\alpha, \beta]\)上的连续函数, 则 \[ g(x) = \int_{\alpha}^{\beta} f(x, y) \,dy \]\([a,b]\)上可微,且 \[ g'(x) = \frac{\partial }{\partial x} \int_{\alpha}^{\beta} f(x, y) \,dy = \int_{\alpha}^{\beta} \frac{\partial f(x,y)}{\partial x} \,dy . \]

定理B.11 (参变积分求导) \(f(x,y)\)\(\frac{\partial f(x,y)}{\partial x}\)都是\([a,b] \times [\alpha, \beta]\)上的连续函数, \(\phi(x)\), \(\psi(x)\)\([a, b]\)上的可微函数且取值于\([\alpha,\beta]\), 则 \[ g(x) = \int_{\phi(x)}^{\psi(x)} f(x, y) \,dy \]\([a,b]\)上可微,且 \[\begin{aligned} g'(x) =& \frac{\partial }{\partial x} \int_{\phi(x)}^{\psi(x)} f(x, y) \,dy \\ =& \int_{\phi(x)}^{\psi(x)} \frac{\partial f(x,y)}{\partial x} \,dy + f(x, \psi(x)) \psi'(x) - f(x, \phi(x)) \phi'(x) . \end{aligned}\]

B.8 向量和矩阵的微分

B.8.1 关于向量的微分

\(f : \mathbb R^p \rightarrow \mathbb R\), 记\(\frac{\partial f(\boldsymbol x)}{\partial \boldsymbol x}\)\(f\)\(p\)个一阶偏导数组成的列向量, 称为\(f\)的梯度, 记一阶偏导数组成的行向量为 \(\frac{\partial f(\boldsymbol x)}{\partial \boldsymbol x^T}\)

\(\frac{\partial^2 f(\boldsymbol x)}{\partial \boldsymbol x \partial \boldsymbol x^T}\)\(f\)的二阶偏导数组成的\(p \times p\)矩阵, 称为\(f\)的海色阵(Hessian)。

\(\boldsymbol a\)\(p\)维列向量,\(A\)\(p \times p\)对称阵, 则 \[\begin{align*} & \frac{\partial (\boldsymbol a^T \boldsymbol x )}{\partial \boldsymbol x} = \boldsymbol a, \quad \frac{\partial (\boldsymbol x^T \boldsymbol a )}{\partial \boldsymbol x} = \boldsymbol a, \\ & \frac{\partial (\boldsymbol x^T A \boldsymbol x)}{\partial \boldsymbol x} = 2 A \boldsymbol x, \\ & \frac{\partial^2 (\boldsymbol x^T A \boldsymbol x)}{\partial \boldsymbol x \partial \boldsymbol x^T} = 2 A . \end{align*}\]

B.8.2 关于矩阵的微分

\(f(\boldsymbol X)\)是以矩阵\(\boldsymbol X = (x_{ij})_{m \times n}\)为自变量的实值函数, 关于各矩阵元素可导, 记\(\frac{\partial f(\boldsymbol X)}{\partial \boldsymbol X}\) 表示\(f\)关于每个元素\(x_{ij}\)的偏导数组成的矩阵, 即 \[ \left( \frac{\partial f(\boldsymbol X)}{\partial x_{ij}} \right)_{m \times n} . \]

性质:

\(\boldsymbol X_{m\times n}\), \[\begin{aligned} & \frac{\partial f(\boldsymbol X)}{\partial \boldsymbol X^T} = \left( \frac{\partial f(\boldsymbol X)}{\partial \boldsymbol X} \right)^T . \\ \end{aligned}\]

\(\boldsymbol X_{m\times n}\)\(\boldsymbol A_{n \times m}\)\[\begin{aligned} & \frac{\partial \text{tr}(\boldsymbol X \boldsymbol A)}{\partial \boldsymbol X} = \frac{\partial \text{tr}(\boldsymbol A \boldsymbol X)}{\partial \boldsymbol X} = \boldsymbol A^T, \end{aligned}\]\(\boldsymbol X_{m\times n}\)\(\boldsymbol A_{m \times n}\)\[\begin{aligned} & \frac{\partial \text{tr}(\boldsymbol X^T \boldsymbol A)}{\partial \boldsymbol X} = \frac{\partial \text{tr}(\boldsymbol A \boldsymbol X^T)}{\partial \boldsymbol X} = \boldsymbol A . \\ \end{aligned}\]

\(\boldsymbol X_{m\times n}\), \(\boldsymbol A_{p\times m}\), \(\boldsymbol B_{n\times p}\), \[\begin{aligned} & \frac{\partial \text{tr}(\boldsymbol A \boldsymbol X \boldsymbol B)}{\partial \boldsymbol X} = \frac{\partial \text{tr}(\boldsymbol B \boldsymbol A \boldsymbol X)}{\partial \boldsymbol X} = \boldsymbol A^T \boldsymbol B^T . \\ \end{aligned}\]

\(\boldsymbol X_{m\times n}\)和对称阵\(\boldsymbol A_{n\times n}\), \[\begin{aligned} & \frac{\partial \text{tr}(\boldsymbol X \boldsymbol A \boldsymbol X^T)}{\partial \boldsymbol X} = 2 \boldsymbol X \boldsymbol A . \end{aligned}\]

\(\boldsymbol X_{m\times n}\), \(\boldsymbol A_{n\times m}\), \(\boldsymbol B_{n\times m}\), \[\begin{aligned} & \frac{\partial \text{tr}(\boldsymbol X \boldsymbol A \boldsymbol X \boldsymbol B)}{\partial \boldsymbol X} = \boldsymbol B^T \boldsymbol X^T \boldsymbol A^T + \boldsymbol A^T \boldsymbol X^T \boldsymbol B^T . \end{aligned}\]

\(\boldsymbol X_{m\times n}\), \(\boldsymbol A_{n\times n}\), \(\boldsymbol B_{m\times m}\), \[\begin{aligned} & \frac{\partial \text{tr}(\boldsymbol X \boldsymbol A \boldsymbol X^T \boldsymbol B)}{\partial \boldsymbol X} = \boldsymbol B^T \boldsymbol X \boldsymbol A^T + \boldsymbol B \boldsymbol X \boldsymbol A . \end{aligned}\]

\(\boldsymbol X_{m\times n}\), \(\boldsymbol B_{m\times m}\), \[\begin{aligned} & \frac{\partial \text{tr}(\boldsymbol X^T \boldsymbol X \boldsymbol B)}{\partial \boldsymbol X} = \boldsymbol X ( \boldsymbol B + \boldsymbol B^T) . \end{aligned}\]

对可逆的\(m\times m\)矩阵\(\boldsymbol X\),有 \[\begin{aligned} \frac{\partial \log \text{det}(\boldsymbol X)}{\partial \boldsymbol X} =& (\boldsymbol X^T)^{-1}, \\ \frac{\partial \text{det}(\boldsymbol X^{-1})}{\partial \boldsymbol X} =& -\frac{1}{\text{det}(\boldsymbol X)} (\boldsymbol X^T)^{-1}, \\ \frac{\partial \text{det}(\boldsymbol X^{-1})}{\partial \boldsymbol X^{-1}} =& -\text{det}(\boldsymbol X) \boldsymbol X^T . \\ \end{aligned}\]

○○○○○○

References

Brockwell, P. J., and R. A. Davis. 1987. Time Series: Theory and Methods. Springer-Verlag.